Chapter 9: Muscles and Chapter 10 Test 3 (BIO 201 SUMMER 2019), A&P Lecture Ch. 8,9,11, Mastering HW Ch 9, Mastering Chapter 10 Activities, Ch. 7 Muscles A&P

Ace your homework & exams now with Quizwiz!

muscle fiber type: functional characteristics

* Speed of contraction: determined by speed in which ATPases split ATP --The two types of fibers are slow and fast * ATP-forming pathways --Oxidative fibers - use aerobic pathways --Glycolytic fibers - use anaerobic glycolysis * These two criteria define three categories: slow oxidative fibers, fast oxidative fibers, and fast glycolytic fibers

treppe: the staircase effect

* increased contraction in response to multiple stimuli of the same strength * contraction increases because: -- there is increasing availability of Ca2+ in the sarcoplasm -- muscle enzyme systems become more efficient because heat is increased as muscle contracts

isometric contraction

* increasing muscle tension but the muscle neither shortens nor lengthens * occurs if the load is greater than the tension the muscle is able to develop ex: trying to pick up a piano

muscle tone

* is the constant, slightly contracted state of all muscles, which does not produce active movements * keeps the muscles firm, healthy, and ready to respond to stimulus

as K+ leaves the cell: (RMP)

* the charge becomes more negative this negative charge attracts K+ back into the cell

muscle fatigue

* the muscle is in a state of physiological inability to contract * occurs when: -- ATP production fails to keep pace with ATP use -- there is a relative deficit of ATP, causing contractions -- lack of ATP means that Ca2+ can't be removed from the sarcoplasm and actin and myosin can't release * lactic acid accumulates in the muscle which lowers the pH * ionic imbalances are present (Na+ and K+) * intense exercise produces rapid muscle fatigue (with rapid recovery) -- Na+ K+ pumps cannot restore ionic imbalances quickly enough * low intensity exercise produces slow-developing fatigue -- SR is damaged and Ca2+ regulation is disrupted

resting membrane potential (RMP)

* the voltage across the cell membrane * this can be measured with very tiny probes; one inside the membrane and one outside * depending on the cell type RMP -- ranges from -5 to -100 m V * all cells are polarized

muscle attach: directly or indirectly

- directly: epimysium of the muscle is fused to the periosteum of the bone - indirectly: CT wrappings extend beyond the muscle as a tendon or aponeurosis - indirect attachments are more common: they are more durable and take up less room on the bone and across the joint

microscopic anatomy of a skeletal muscle fiber

- each fiber is a long, cylindrical cell with multiple nuclei just beneath the sarcolemma - fibers are up to hundreds of centimeters long - each cell is produced by fusion of embryonic cells - sarcoplasma has numerous glycosomes and lots of myoglobin

3 connective sheaths of skeletal muscle

- endomysium - perimysium - epimysium Think of a 3 pack of gum: - endomysium is the wrapper around each piece of gum - perimysium is the package of one gum pack - epimysium is the plastic holding all 3 gum packages together

myofilaments: Thin filaments

- extend across the I band and partway into the A band

Myofilaments: Thick Filaments

- extend the entire length of an A band - H-zone is only thick filaments

The following is a list of the events that occur during a muscle contraction. What is the correct sequence of these events? 1. Myosin cross-bridges bind to the actin. 2. The free myosin head splits ATP. 3. Calcium ion is released from the sarcoplasmic reticulum. 4. The myosin head pivots toward the center of the sarcomere. 5. Calcium ion binds to troponin. 6. The myosin head binds an ATP molecule and detaches from the actin.

3. calcium ion is released from the sarcoplasmic reticulum 5. calcium ion binds to troponin 1. myosin cross-bridges bind to the actin 4. the myosin head pivots toward the center of the sarcomere 6. the myosin head binds an ATP molecule and detaches from the actin 2. the free myosin head splits ATP

The cross bridge cycle is a series of molecular events that occur after excitation of the sarcolemma. What is a cross bridge?

A myosin head bound to actin

Which selection best describes the initial event in contraction?

Myosin heads bind to the newly exposed myosin-binding sites on actin to form cross bridges. -Contraction begins when myosin binds to actin and forms cross bridges.

Arrange the steps of the contraction cycle in the correct order from left to right. The myosin heads bind to actin active sites Calcium ions arrive within the zone of overlap The free myosin head "recocks". ADP and P are released calcium ions bind to troponin. active sites on actin are exposed ATP binds to the myosin head, breaking the link The myosin head pivots (power stroke). ADP and P are released

Step 1: Calcium ions arrive within the zone of overlap Step 2: calcium ions bind to troponin. active sites on actin are exposed Step 3: The myosin heads bind to actin active sites Step 4: The myosin head pivots (power stroke). ADP and P are released Step 5: ATP binds to the myosin head, breaking the link Step 6: The free myosin head "recocks". ADP and P are released

QUESTION 1: Inadequate calcium in the neuromuscular junction would directly affect which of the following processes? (question 17) a. Breakdown of acetylcholine by acetylcholinesterase b. Release of acetylcholine from the synaptic vesicles c. Conduction of action potentials by the motor neuron d. Depolarization of the junctional folds

b. Release of acetylcholine from the synaptic vesicles

QUESTION 1: Which of the following statements is true? Choose the best answer. a. The acetylcholine (ACh) receptors are located on the myofibril b. The neurotransmitter is stored in the presynaptic motor neuron c. The sarcolemma is the presynaptic membrane d. Acetylcholinesterase (AChE) is stored in vesicles within the presynaptic motor neuron

b. The neurotransmitter is stored in the presynaptic motor neuron

Muscle fatigue occurs because of a buildup of __________ and a(n) __________ in pH. a. creatine phosphate; increase b. lactic acid; decrease c. creatine phosphate; decrease d. lactic acid; increase

b. lactic acid; decrease

QUESTION 1: Bacteria that cause tetanus release a neurotoxin. Which of the following components of the neuromuscular junction (NMJ) does this neurotoxin affect directly? a. sarcolemma b. motor neurons c. epimysium d. myofilaments

b. motor neurons

Each skeletal muscle fiber is controlled by a motor neuron at a single a. sarcomere b. neuromuscular junction c. synaptic cleft d. transverse tubule e. synaptic knob

b. neuromuscular junction

Minimum contraction

contraction caused by threshold stimulus

Which of the following is NOT a function of skeletal muscles?

contraction of the heart

Period of contraction of muscle twitch

cross bridges form and cycling occurs; muscle shortens

During anaerobic glycolysis, a. carbohydrate is metabolized b. ATP is produced c. oxygen is not consumed d. pyruvic acid is produced e. All of the answers are correct

e. All of the answers are correct

When neurotransmitter molecules bind to receptors in the plasma membrane of the receiving neuron,

ion channels in the plasma membrane of the receiving neuron open.

As you unsuccessfully attempt to lift the 300-pound weights off the ground, you are demonstrating

isometric contraction

gastrocnemius muscle

plantar flexes foot and leg

insertion and origin

points where muscles attach to bones, name used depends on mvmt of the bone

When calcium ions enter the synaptic terminal,

they cause vesicles containing neurotransmitter molecules to fuse to the plasma membrane of the sending neuron.

What is name given to the regularly spaced infoldings of the sarcolemma?

transverse or T tubules

What is name given to the regularly spaced infoldings of the sarcolemma?

transverse or T tubules -Yes! T tubules penetrate a skeletal muscle fiber and provide a pathway for excitation into the interior.

Muscle Metabolism: Energy for Contraction (ATP REGENERATION)

* ATP is a direct source of energy for muscle contraction but very little ATP is stored in muscle * once it is used up (in 4-6 seconds), it is regenerated by: -- the interaction of ADP with creatine phosphate (CP) -- anaerobic glycolysis -- aerobic respiration

creatine phosphate CP (ATP regeneration)

* CP is high energy molecules stored in muscles * CP + ADP --> CREATINE + ATP * provides enough energy for about 10 more seconds

ultrastructure of myofilaments: thin filaments

* Thin filaments are chiefly composed of the protein actin * Each actin molecule looks like 2 strands of pearl twisted together and the pearls are G action * tropomyosin is a protein that spirals around the actin and blocks the active sites where myosin binds * troponin attaches tropomyosin to the actin and regulates contraction

muscle twitch

* a muscle twitch is the response of a muscle to a single, brief threshold stimulus * a single stimulus results in a single contractile response

two types of muscle contraction

* isometric contraction * isotonic contraction

contraction of muscle

* muscle tension * load

anaerobic glycolysis (muscle metabolism)

* when muscle contractile activity reaches 70% of maximum: -- bulging muscles compress blood vessels -- oxygen delivery is impaired -- pyruvic acid is converted into lactic acid (not good: fatigue quicker and sore muscles)

what do all of these light and dark areas mean in myofibrils?

- all of these bands are caused by the arrangement of myofilaments

lactic acid

-- is toxic to molecules in high levels -- cause fatigue and muscle soreness

striations and sarcomeres (in myofibrils)

-- with a light microscope we see light and dark bands

Assume you have a membrane with only potassium leakage channels. The RMP is -90mV. Predict the RMP if we add Na+ leakage channels.The most likely RMP value of Na+ is __________.

-70 mV

3 phases of muscle twitch

1) latent period 2) period of contraction 3) period of relaxation

2 types of gated channels

1. chemically (ligand) gated 2. voltage gated

Put the following structures in order from superficial to deep. 1. muscle fiber 2. perimysium 3. myofibril 4. fascicle 5. endomysium 6. epimysium

6. epimysium 2. perimysium 4. fascicle 5. endomysium 1. muscle fiber 3. myofibril

After a power stroke, the myosin head must detach from actin before another power stroke can occur. What causes cross bridge detachment?

ATP binds to the myosin head

Which of the choices below correctly describes how an action potential generated at the neuromuscular junction (NMJ) is converted to excitation in the muscle fiber?

An action potential in the motor neuron causes ACh to be released into the synaptic cleft. Binding of ACh to sarcolemma receptors initiates graded potentials.

Calcium ions couple excitation of a skeletal muscle fiber to contraction of the fiber. Where are calcium ions stored within the fiber?

Calcium ions are stored in the sarcoplasmic reticulum

What specific event triggers the uncovering of the myosin binding site on actin?

Calcium ions bind to troponin and change its shape.

Which of the following is most directly responsible for the coupling of excitation to contraction of skeletal muscle fibers?

Calcium ions. -Yes! Action potentials propagating down the T-tubule cause a voltage-sensitive protein to change shape. This shape change opens calcium release channels in the sarcoplasmic reticulum, allowing calcium ions to flood the sarcoplasm. This flood of calcium ions is directly responsible for the coupling of excitation to contraction in skeletal muscle fibers.

What element of fibrous joints gives the joint its ability to resist stretching and control the amount of movement at the joint?

Collagen fibers

What changes occur to voltage-gated Na+ and K+ channels at the peak of depolarization?

Inactivation gates of voltage-gated Na+‎ channels close, while activation gates of voltage-gated K+‎ channels open. -Yes! Closing of voltage-gated channels is time dependent. Typically, the inactivation gates of voltage-gated Na+‎ channels close about a millisecond after the activation gates open. At the same time, the activation gates of voltage-gated K+‎ channels open.

How does the shape change of these proteins lead to contraction?

It allows calcium to exit the sarcoplasmic reticulum and enter the cytosol

People with multiple sclerosis (MS) experience many challenging symptoms. Which statement best explains one cause of these difficulties?

Loss of oligodendrocytes in the central nervous system results in absence of the myelin sheath of neuronal axons, impairing rapid saltatory conduction. -The oligodendrocytes of the central nervous system provide axonal myelin sheaths, which increases the speed of conduction of action potentials. The loss of myelin inhibits this conduction in MS and other demyelinating disorders.

myofilaments: M-lines

M-lines appear darker due to the presence of an anchoring protein

Which of the following does NOT occur in the sacromere during muscle contraction?

The Z-lines move further apart

Which mechanism allows the rabies virus to gain access to the central nervous system (CNS)?

The rabies virus uses retrograde movement along the neuronal axon. -Once the rabies virus enters the tissue via a bite, the viral particles use the normal retrograde movement along the axons of the peripheral nervous system to move "upstream" to the CNS.

If troponin is a component of both cardiac and skeletal muscle, why is an elevated plasma troponin level useful in diagnosing myocardial damage?

The subunits of the troponin in cardiac muscle are unique to heart muscle. -Although both types of striated muscle contain troponin, the subunits found in cardiac muscle are different from those in skeletal muscle. The laboratory test used to diagnose cardiac muscle damage is specific for those subunits.

A(n) ________ can be described as a broad tendinous sheet. a. aponeurosis b. interstitium c. fasciae e. retinaculum d. tympanum

a. aponeurosis

Muscles that move the eyeball have ________ fibers. a. fast b. circular c. slow d. intermediate e. All of the answers are correct.

a. fast

Large-diameter, densely packed myofibrils, large glycogen reserves, and few mitochondria are characteristics of a. fast fibers b. fatty muscles c. red muscles d. intermediate fibers e. slow fibers.

a. fast fibers

QUESTION 2: Which of the following processes produces molecules of ATP and has two pyruvic acid molecules as end products? a. glycolysis b. Krebs cycle and oxidative phosphorylation c. hydrolysis of creatine phosphate

a. glycolysis

Most of the ATP needed by muscle cells is provided through

aerobic metabolism

What is the function of the structure indicated by the arrow? (question 36) a. making of energy (ATP); "power house" of the cell b. distribute action potentials throughout the interior of the skeletal muscle cell c. houses the genetic material (DNA) of the cell d. storage of calcium

b. distribute action potentials throughout the interior of the skeletal muscle cell

Muscle fibers differ from "typical cells" in that muscle fibers a. are very small b. have many nuclei c. lack mitochondria d. lack a plasma membrane e. have large gaps in the cell membrane

b. have many nuclei

After death, muscle fibers run out of ATP and calcium begins to leak from the sarcoplasmic reticulum into the sarcoplasm. This results in a condition known as a. tetany b. rigor mortis c. depolarization d. treppe e. oxygen debt

b. rigor mortis

The plasma membrane of a skeletal muscle fiber is called the a. sarcoplasm b. sarcolemma c. sarcoplasmic reticulum d. sarcomere e. sarcosome

b. sarcolemma

The repeating unit of a skeletal muscle fiber is the a. myofibril b. sarcomere c. myofilament d. sarcolemma e. sarcoplasmic reticulum

b. sarcomere

Which of the following statements correctly describes the structure (band or line) of the sarcomere indicated by the arrow? (question 41) a. the point of connection for adjacent thick filaments b. the region of the sarcomere that contains only thin filaments c. the region of the resting sarcomere that only contains thick filaments d. the boundary between adjacent sarcomeres

b. the region of the sarcomere that contains only thin filaments

What causes the myosin head to disconnect from actin?

binding of ATP

Which of the following is a way that muscle fibers get energy needed for contraction? a. Aerobic exercise is beneficial to a muscle because it increases the muscle's ability to acquire greater amounts of glucose from the blood for ATP production b. During moderate exercise, such as jogging on the treadmill, pyruvate and hydrogen ions accumulate causing muscle fatigue c. During peak activity levels, glycolysis is the only pathway by which ATP can be produced to supply energy to the muscle d. When a muscle is contracting during peak activity, it quickly uses up stored creatine phosphate which was produced during moderate exercise. e. Because energy demands are low in a resting muscle, a resting muscle is able to meet its ATP requirement through glycolysis

c. During peak activity levels, glycolysis is the only pathway by which ATP can be produced to supply energy to the muscle

Myasthenia gravis is a disease resulting from an autoimmune attack on the ACh receptors of the motor end plate. Binding of antibodies to the ACh receptors results in generalized muscle weakness that progresses as more ACh receptors are destroyed. Which of the following medications would help alleviate the muscle weakness? a. a drug that blocks the release of acetylcholine (botulinum toxin) b. a drug that binds to the acetylcholine receptor and prevents it from opening (curare) c. a drug that binds to and inactivates acetylcholinesterase (neostigmine) d. a drug that prevents acetylcholine from being loaded into synaptic vesicles (vesamicol)

c. a drug that binds to and inactivates acetylcholinesterase (neostigmine)

At what point during excitation contraction coupling does exocytosis play a role? a. when ATP splits into ADP and P on the free myosin head b. during calcium ion reuptake into the sarcoplasmic reticulum c. during acetylcholine release from the synaptic terminal d. when sodium channels open up on the motor end plate e. when the action potential surges through the T-tubules

c. during acetylcholine release from the synaptic terminal

The sequence of processes that links the action potential to contraction is called a. cross bridge formation. b. neuromuscular junction c. excitation-contraction coupling d. action potential propagation e. sliding filament theory

c. excitation-contraction coupling

A fascicle is a a. layer of connective tissue that separates muscle from skin b. collection of myofibrils in a muscle fiber c. group of muscle fibers that are encased in the perimysium d. group of muscle fibers and motor neurons e. group of muscle fibers that are all part of the same motor unit

c. group of muscle fibers that are encased in the perimysium

Which of the following types of muscle fibers are best adapted for prolonged contraction such as standing all day? a.intermediate fibers b. uninucleated fibers c. slow fibers d. striated fibers e. fast fibers

c. slow fibers

Which of the following hormones directly stimulates growth of muscle tissue, leading to increased muscle mass? a. epinephrine b. thyroid hormone c. testosterone d. parathyroid hormone e. calcitonin

c. testosterone

QUESTION 7: An action potential in the muscle fiber causes __________. a. the release of acetylcholine into the synaptic cleft b. acetylcholine to bind to receptors on the motor end plate c. the muscle fiber to contract d. acetylcholinesterase to break down acetylcholine

c. the muscle fiber to contract

orbicularis oculi muscle

closes eyelid

masseter muscle

closes jaw

orbicularis oris muscle

closes lip (kissing and whistling muscle)

The distance between Z discs ________ during muscle contraction.

decreases

An action potential is self-regenerating because __________.

depolarizing currents established by the influx of Na+‎ flow down the axon and trigger an action potential at the next segment -Yes! The Na+‎ diffusing into the axon during the first phase of the action potential creates a depolarizing current that brings the next segment, or node, of the axon to threshold.

tibialis anterior muscle

dorsiflexes and inverts foot

The dense layer of connective tissue that surrounds an entire skeletal muscle is the a. endomysium b. perimysium c. tendon d. fascicle e. epimysium

e. epimysium

Which of the following allows muscles to return to their original shape during relaxation? a. elastic forces b. opposing muscle contractions c. gravity d. myosin binding e. gravity, opposing muscle contractions, and elastic forces

e. gravity, opposing muscle contractions, and elastic forces

The type of contraction in which the muscle fibers do not shorten is called a. isotonic b. tetany c. concentric d. treppe e. isometric

e. isometric

During the time when the action potential moves through the sarcolemma a muscle twitch is in a. contraction phase b. stimulus phase c. relaxation phase d. isotonic period e. latent period

e. latent period

You try to pick up an object and discover that it is much heavier than you expected. Which process must occur in the muscle to increase tension so you can pick up the object? a. treppe b. isotonic contraction c. complete tetanus d. wave summation e. recruitment

e. recruitment

The term used to describe muscular growth in response to usage is

hypertrophy

What structures are found in the cardiac muscles?

intercalated disc

Sarcolemma

muscle plasma membrane

the muscle is relaxed when?

myosin is blocked from action

ach is?

neurotransmitter found in synaptic vesicles of the axonal terminal

muscle response to varying stimuli

notice that there is an increase in strength of the contraction, this is because we're adding contractions onto existing contractions

In multiple sclerosis, the cells that are the target of an autoimmune attack are the _________.

oligodendrocyte -Oligodendrocytes are a type of neuroglial cell that function to form the myelin sheath around the axons of neurons within the central nervous system. These are the glial cells targeted by the immune system in people who have MS.

voltage gated channels

open in response to a change in charge * think of a garage door opener, you need an electrical signal not a actual key

chemically (ligand) gated channels

open in response to something binding to them ex: acetylcholine receptors in the motor end plate * think of a lock on a door, you need a key to open it

Let's consider a scenario in which the resting membrane potential changes from −70 mV to +70 mV, but the concentrations of all ions in the intracellular and extracellular fluids are unchanged. Predict how this change in membrane potential affects the movement of Na+. The electrical gradient for Na+ would tend to move Na+ __________ while the chemical gradient for Na+ would tend to move Na+ __________.

out; in =If the inside of the cell is positively charged compared to the outside, the electrical gradient will favor movement of positively charged sodium ions out of the cell.

Muscle tissue does NOT ________.

produce blood cells

frontalis muscle

raises eyebrows, wrinkles forehead

temporalis muscle

raises mandible and closes jaw

trapezius muscle

raises, retracts, and rotates the shoulder

a cell becoming more negative =

repolarization

Sternocleidomastoid muscle

rotates head

Which factor is likely to most likely to protect against shoulder dislocation?

rotator cuff muscles -Tendons of the rotator cuff muscles (supraspinatus, infraspinatus, teres minor, and subscapularis) surround and provide stability to the shoulder joint.

Slow oxidative muscle fibers are best suited for ________.

running a marathon

What structure is the functional unit of contraction in a skeletal muscle fiber?

sarcomere

Which muscle fiber type is best suited for endurance activities, such as long-distance jogging?

slow oxidative fibers

Nonstriated, involuntary muscle is

smooth

complete tetanus contraction

smooth and sustained contraction ex: holding a book or holding your arm out

Binding of calcium to calmodulin is a step in excitation-contraction coupling of ________ cells.

smooth muscle

The first step toward generating a skeletal muscle contraction is ________.

stimulation of the muscle by a nerve ending

Perimysium

surrounds group of muscle fibers called fascicles

What factor does not contribute to the strength and stability of a synovial joint?

synovial fluid

The action potential on the muscle cell leads to contraction due to the release of calcium ions. Where are calcium ions stored in the muscle cell?

terminal cesterns (cisternae) of the sarcoplasmic reticulum

Na+ does try to move into the cell: (RMP)

the membrane is only slightly permeable to Na+

eccentric contraction (type of isotonic contraction)

the muscle contracts as it lengthens

concentric contraction (type of isotonic contraction)

the muscle shortens and does work

Acetylcholine binds to its receptor in the sarcolemma and triggers __________.

the opening of ligand-gated cation channels

2 kinds of myofilaments

thick and thin

Drag the labels onto the diagram to identify the four different types of effects of repeated stimulation on a skeletal muscle. (question 19)

treppe wave summation incomplete tetanus complete tetanus

block(s) binding sites on actin

tropomyosin

Which protein inhibits interaction between actin and myosin to prevent skeletal muscle contraction; and which ions remove the inhibition?

tropomyosin; calcium ions

The type of muscle found in the walls of most hollow organs is ________.

unitary smooth muscle

When an action potential arrives at the axon terminal of a motor neuron, which ion channels open?

voltage-gated calcium channels

the cell membrane is impermeable to: (RMP)

* Cl- (chloride) because there is no pump allowing them to move * A- (protein anions) because theyre too big

anaerobic metabolism (ATP regeneration)

* forms 2 ATP/glucose molecules * no oxygen used in the pathway * forms pyruvic acid -- pyruvic acid can enter the aerobic pathway or be converted to lactic acid * provides about 45 more seconds of energy

nerve and blood supply of skeletal muscle

-Each muscle is served by one nerve, an artery, and one or more veins -Each skeletal muscle fiber is supplied with a nerve ending that controls contraction

dark bands

A bands

Click and drag the description of the part of the excitation-contraction coupling to the correct order of occurrence from left to right. Do not overlap any steps.

Coupling begins with an action potential after the arrival of ACh and graded potentials, and it ends before cross bridge cycling.

When does cross bridge cycling end?

Cross bridge cycling ends when sufficient calcium has been actively transported back into the sarcoplasmic reticulum to allow calcium to unbind from troponin.

A person dies, and within hours, the skeletal muscles develop a locked contraction known as rigor mortis. Calcium ions leak from the sarcoplasmic reticulum into cytoplasm. From your knowledge of cross bridge cycling, what best explains this rigor?

Cross bridge detachment cannot occur. Detachment requires ATP, which is produced only during life. -After ATP attaches to the myosin head, the bond between actin and myosin is weakened and the cross bridge breaks.

Which of the following is NOT an effect that exercise will have on muscle tissue? A. higher concentration of glycolytic enzymes B. increase in the number of myofibrils C. increase in the amount of tension produced when the muscle contracts D. an increase in the amount of fibrous connective tissue

D. an increase in the amount of fibrous connective tissue

QUESTION 4: Drag the measurements of different regions of the sarcomere to their respective locations (bins) to describe what happens during the sliding filament mechanism of sarcomere shortening. BIN 1: decreasing in length or distance BIN 2: no change in length or distance BIN 3: increasing in length or distance Length of A band Length if I band Length of H zone Length of thin filament length of thick filament distance between Z discs

Decreases in length or distance 1.) Length of I band 2.) Length of H zone 3.) Distance between Z discs No change in length or distance 1.) Length of A band 2.) Length of thick filament 3.) Length of thin filament

T/F- Isometric contraction leads to movement of a load.

False

T/F- Cardiac muscle makes most of its ATP via anaerobic pathways.

False (aerobic)

QUESTION 4: Drag the appropriate events of excitation-contraction coupling to their respective locations (bins). Tropomyosin shifts position Motor neuron signals muscle fiber contraction Calcium ions are released Sodium channels open along sarcolemma Myosin forms cross bridges with actin Action potential is conducted along T tubule Calcium ions bind to troponin

First bin: Motor neuron signals muscle fiber contraction Sodium channels open along sarcolemma Second bin: Calcium ions are released Action potential is conducted along T tubule Third bin: Myosin forms cross bridges with actin Tropomyosin shifts position Calcium ions bind to troponin

wave summation contraction

Frequently delivered stimuli (muscle does not have time to completely relax) increases contractile force

Which of the following is ranked from smallest to largest?

H-band, sacromere, myofibril, muscle fiber, fascicle, muscle

light bands

I bands - think 'illumination'

The smallest contractile unit of a muscle fiber is ________.

the sarcomere

What is the relationship between the number of motor neurons recruited and the number of skeletal muscle fibers innervated?

Typically, hundreds of skeletal muscle fibers are innervated by a single motor neuron.

QUESTION 3: Identify the regions of the sarcomere. Note that labels may be used more than once.

View image on question 13

The characteristic muscle stiffness associated with rigor mortis is due to the inability of myosin filaments to detach from the active site on actin filaments. What molecule is essential for this detachment? a. ATP b. calcium c. acetylcholine d. acetylcholinesterase

a. ATP

Triggering of the muscle action potential occurs after a. acetylcholine binds to chemically-gated channels in the motor end plate b. the action potential jumps across the neuromuscular junction c. acetylcholinesterase is released from synaptic vesicles into the synaptic cleft d. calcium ion binds to channels on the motor end plate e. Any of these can produce an action potential in the muscle cell

a. acetylcholine binds to chemically-gated channels in the motor end plate

QUESTION 5: What causes the vesicles inside a neuron to fuse with the plasma membrane? a. an action potential in the neuron b. acetylcholine being broken down by acetylcholinesterase c. an action potential in the muscle fiber d. acetylcholine binding to acetylcholine receptors

a. an action potential in the neuron

When acetylcholine binds to receptors at the motor end plate, the sarcolemma becomes a. more permeable to sodium ions b. less permeable to potassium and sodium ions c. less permeable to sodium ions d. less permeable to potassium ions e. more permeable to calcium ions

a. more permeable to sodium ions

What is the function of the structure indicated by the arrow? (question 34) a. storage of calcium b. houses the genetic material (DNA) of the cell c. distribute action potentials throughout the interior of the skeletal muscle cell d. making of energy (ATP); "power house" of the cell

a. storage of calcium

deltoid muscle

abducts, flexes, and extends arm

QUESTION 2: Which of the following events directly occurs due to an action potential generated on the motor neuron? Choose the best answer. a. ACh binds to ACh receptors b. ACh is released at the synapse.Na rushes into the sarcolemma c. AChE breaks down ACh in the synaptic cleft d. An action potential is generated on the muscle fiber.

b. ACh is released at the synapse.Na rushes into the sarcolemma

QUESTION 3: The post-synaptic reaction on the sarcolemma is short in duration. Which of the following explanations describes why the reaction of the sarcolemma is short in duration? Select all that are correct. a. Muscle tissue has a smaller membrane potential than nervous tissue b. AChE acts to breakdown the ACh in the synaptic cleft c. ACh diffuses across the synaptic cleft toward the sarcolemma d. ACh diffuses out of the synaptic cleft e. The ACh receptors respond only for a short period of time.

b. AChE acts to breakdown the ACh in the synaptic cleft d. ACh diffuses out of the synaptic cleft

QUESTION 1: What causes the release of calcium ions into the sarcoplasm from the terminal cisterns? (question 15) a. The change in the shape of troponin b. An action potential traveling along the t tubule c. The binding of myosin heads to actin d. Acetylcholine entering the sarcoplasm of the muscle fiber

b. An action potential traveling along the t tubule

A muscle cell runs out of ATP. Even though these are cyclic reactions, what step of the cross bridge cycle given is most directly inhibited or terminated?

cross bridge detachment -When ATP is added to the myosin, myosin detaches from actin.

What can the nervous system do to increase muscle tension? a. increase stimulation frequency b. increase the number of active motor units c. recruit larger motor units d. All of the listed responses can increase muscle tension

d. All of the listed responses can increase muscle tension

biceps brachii muscle

flexes elbow joint; supinates forearm

Thick myofilaments are made of ________.

myosin

The generation of an action potential in a neuron requires the presence what type of membrane channels? Select the best answer.

voltage-gated channels

maximum stimulus

weakest stimulus that causes a maximum response

threshold stimulus

weakest stimulus that causes a response

arrangement of the filaments in a sarcomere

Longitudinal section within one sarcomere - thick: myosin - thin: actin

Botulism is caused by a bacterial toxin that prevents the release of ACh at the axon terminals. What happens as a result?

Muscles are not capable of contracting.

The neuromuscular junction is a well-studied example of a chemical synapse. Which of the following statements describes a critical event that occurs at the neuromuscular junction?

Acetylcholine is released by axon terminals of the motor neuron.

Action potentials (nerve impulses) are changes in the membrane potential that, once started, will affect the entire excitable membrane. The first action potential is usually generated at the initial segment of the neuron's axon. This activity will test your understanding of the sequence of events that occur at the membrane of the initial segment of the axon during generation of an action potential. Place the events involved in generation of an action potential in the correct order of occurrence from left to right.

Action potentials are all-or-nothing events. Once an action potential is generated at the initial segment, it will continue to propagate (move) along the entire length of the axon.

Consider how the action potential that initiates contraction is delivered to the muscle cell. Which of the choices below correctly describes how an action potential generated at the neuromuscular junction (NMJ) is converted to excitation in the muscle fiber?

An action potential in the motor neuron causes ACh to be released into the synaptic cleft. Binding of ACh to sarcolemma receptors initiates graded potentials. -Yes! An action potential traveling down a motor neuron arrives at the axon terminal and causes exocytosis of the neurotransmitter ACh into the synaptic cleft. ACh diffuses into the synaptic cleft, binds to the receptor proteins on the junctional folds of the muscle sarcolemma (motor end plate), and initiates graded potentials. These graded potentials sum to an action potential, thus initiating excitement of the muscle fiber.

Part complete The concentrations of which two ions are highest outside the cell?

Na+ and Cl−

Which of the following joint problems is the result of an autoimmune problem?

Rheumatoid arthritis (RA) -In RA, the body's immune system attempts to destroy the tissues in the joints in an apparent case of "mistaken identity."

Epimysium

CT that surrounds the entire muscle

A triad is composed of a T-tubule and two adjacent terminal cisternae of the sarcoplasmic reticulum. How are these components connected?

A series of proteins that control calcium release.

isotonic contraction and its 2 types

* the muscle changes in length and moves the load * two types of isotonic contractions: -- eccentric -- concentric

aerobic respiration (ATP regeneration)

* uses oxygen to breakdown glucose completely * produces alot of ATP: -- 36 ATP/glucose * glucose + O2 --> CO2 + H2O + ATP * provides the most ATP

How does the myosin head obtain the energy required for activation?

The energy comes from the hydrolysis of ATP.

QUESTION 1: How is delayed-onset muscle soreness (DOMS) different from muscle fatigue? a. Delayed-onset muscle soreness occurs a day or more after the physical exertion b. Muscle performance is impaired c. There may be damage to the sarcolemma e. Delayed-onset muscle soreness involves pain

a. Delayed-onset muscle soreness occurs a day or more after the physical exertion

when the mvmt of K+ out of the cell equals the mvmt into the cell, the charge (RMP) is:

about -70 m V

What causes the release of calcium from the terminal cisternae of the sarcoplasmic reticulum wihtin a muscle cell?

arrival of an action potential

EGTA is a substance that binds calcium ions. Imagine an experimental setup with a motor neuron and a muscle fiber. Stimulation of the motor neuron causes contraction of the muscle fiber through activity at the neuromuscular junction and excitation-contraction coupling. Now, inject the muscle fiber with EGTA. Which of the following effects would EGTA have on excitation-contraction coupling after the neuron releases acetylcholine at the neuromuscular junction? a. It would prevent the junctional folds from triggering an action potential b. It would prevent myosin from forming cross bridges with actin c. It would prevent conduction of an action potential along the sarcolemma d. It would prevent the release of calcium ions from the terminal cisterns

b. It would prevent myosin from forming cross bridges with actin

In response to action potentials arriving along the transverse tubules, the sarcoplasmic reticulum releases a. sodium ions b. calcium ions c. potassium ions d. hydrogen ions e. acetylcholine

b. calcium ions

QUESTION 1: What is the type of chemical reaction used to rebuild ADP into ATP? a. hydrolysis b. dehydration synthesis c. rehydration synthesis

b. dehydration synthesis

A muscle producing almost peak tension during rapid cycles of contraction and relaxation is said to be in a. recruitment b. incomplete tetanus c. wave summation d. treppe e. complete tetanus

b. incomplete tetanus

What area of the thick filament binds to actin once actin's binding sites are exposed? (question 65) a. hinge b. tropomyosin c. troponinmyosin d. cross-bridge (head)

d. cross-bridge (head)

QUESTION 3: What is the synaptic cleft? a. the region of the neuron containing synaptic vesicles b. the border between the motor end plate and the sarcolemma c. the step where acetylcholinesterase (AChE) breaks down, or cleaves, acetylcholine d. the space between the synaptic terminal and the motor end plate

d. the space between the synaptic terminal and the motor end plate

When calcium ion binds to troponin, a. actin heads will bind to myosin b. myosin shortens c. muscle relaxation occurs d. tropomyosin rolls away from the active site e. active sites on the myosin are exposed

d. tropomyosin rolls away from the active site

Cross bridge detachment is caused by ________ binding to the myosin head. a. acetylcholine b. calcium c. acetylcholinesterase d. magnesium e. ATP

e. ATP

Spinal reflexes account for muscle tone by:

* activating one motor unit and then another, so muscle fibers take turns and don't become fatigued * responding to activation of stretch receptors in muscles and tendons ex: helps keep balance and standing up right

glycosomes

- stored glycogen - broken down to glucose=energy

Put the following events of the contraction cycle in the order in which they occur. 1. Cross-bridge detachment 2. Cross-bridge formation 3. Active site exposure 4. Myosin reactivation 5. Calcium ions bind troponin 6. Myosin head pivoting

5. calcium ions bind troponin 3. active site exposure 2. cross-bridge detachment 6. myosin head pivoting 1. cross-bridge detachment 4. myosin reactivation

Which of the following best describes the role of acetylcholinesterase molecules at the neuromuscular junction?

Acetylcholinesterase breaks down ACh, which allows chemically gated ion channels to close.

Which choice best characterizes K+ leakage channels?

transmembrane protein channels that are always open to allow K+ to cross the membrane without the additional input of energy

Which best describes the initial event in contraction?

Myosin heads bind to the newly exposed myosin-binding sites on actin to form cross bridges.

The small space between the sending neuron and the receiving neuron is the

Synaptic Cleft -The synaptic cleft is the small space between the sending neuron and the receiving neuron.

Since each myofibril is attached at either end of the muscle fiber, when sarcomeres shorten, the muscle fiber a. shortens b. strengthens c. weakens d. pulls from the middle e. lengthens

a. shortens

What causes the release of calcium ions into the sarcoplasm from the terminal cisterns?

an action potential traveling along the t tubule

A joint held together by fibrocartilage would be classified as a __________ joint.

cartilaginous -Cartilaginous joints are held together by cartilage. Fibrocartilage holds together symphyses.

The role of acetylcholinesterase is to a. bind to ligand gated sodium channels b. activate acetylcholine c. release acetylcholine into the synaptic cleft d. break down acetylcholine into acetate and choline components e transport acetylcholine across the synaptic cleft

d. break down acetylcholine into acetate and choline components

QUESTION 2: Activities involving eccentric contractions more commonly lead to delayed-onset muscle soreness than concentric or isometric contractions. Which of the following activities is thus a candidate for delayed-onset muscle soreness? a. pushing against the wall in the gym b. flexion of the biceps brachii c. carrying a bowling ball d. extension of the biceps brachii

d. extension of the biceps brachii

Fast fibers a. have low resistance to fatigue b. have many mitochondria c. rely on aerobic metabolism d. have low resistance to fatigue and quick twitches e. have twitches with a very brief contraction phase.

d. have low resistance to fatigue and quick twitches

What is the type of chemical reaction used to rebuild ADP into ATP?

dehydration synthesis

The sliding filament model of contraction states that ________.

during contraction the thin myofilaments slide past the thick myofilaments so that the actin and myosin myofilaments overlap to a greater degree

Which of the following is a recognized function of skeletal muscle? a. guard body entrances and exits b. produce movement c. maintain body temperature d. maintain posture e. all of these are correct

e. all of these are correct

When a muscle is stimulated repeatedly at a high rate, the amount of tension gradually increases to a steady maximum tension. This state of maximum tension is called a. recruitment b. incomplete tetanus c. wave summation d. a twitch e. complete tetanus

e. complete tetanus

Endomysium

fine sheath of CT surrounding each muscle fiber

muscle tension

force exerted by contracting muscle

Which of the following processes produces molecules of ATP and has two pyruvic acid molecules as end products?

glycolysis

Unlike skeletal muscle tissue, smooth muscle tissue

is involuntary, lacks sarcomeres, and is not striated

Complete the following sentence. The operation of the Na+−K+ ATPase pump __________.

moves 3 Na+ to the ECF and 2 K+ to the cytoplasm -The pumps move more Na+ than K+, which counteracts the effect of the membrane's being more permeable to K+ (more K+ leak channels than Na+).

QUESTION 1: The neuromuscular junction is a connection between a neuron and a __________. a. vesicle b. synaptic terminal c. myofibril d. muscle fiber

d. muscle fiber

During the recovery period following exercise, all of the following are true except a. heat is generated b. lactic acid is removed from muscle cells c. oxygen is consumed at above the resting rate d. muscle fibers are unable to contract e. the muscle actively produces ATP

d. muscle fibers are unable to contract

The increase in muscle tension that is produced by increasing the number of active motor units is called a. treppe b. complete tetanus c. wave summation d. recruitment e. incomplete tetanus.

d. recruitment

Question 2 The end of a neuron, where acetylcholine-filled vesicles are located, is called the __________. a. acetylcholine receptor b. motor end plate c. synaptic cleft d. synaptic terminal

d. synaptic terminal

The connective tissue sheaths of skeletal muscle, in order from internal to external, are the ________.

endomysium, perimysium, and epimysium

Conduction of an action potential along the sarcolemma depends upon

diffusion of sodium ions through voltage-gated channels

A molecule that carries information across a synaptic cleft is a

neurotransmitter -Neurotransmitter molecules carry information across a synaptic cleft.

Heat Production During Muscle Activity

-Only 40% of the energy released in muscle activity is useful as work -The remaining 60% is given off as heat -Dangerous heat levels are prevented by radiation of heat from the skin and sweating

characteristics of muscle tissue

1. Excitability or irritability: ability to receive and respond to stimuli 2. Contractility: ability to shorten forcibly 3. Extensibility: ability to be stretched or extended beyond resting length 4. Elasticity: ability to recoil and resume the original resting length after being stretched

Botulism and myasthenia gravis are conditions that cause muscle weakness. Which of these statements is NOT true?

Both conditions are caused by an inability of the body to produce adequate acetylcholine. -Although both conditions affect normal activity at the neuromuscular junction, failure to produce acetylcholine is not the mechanism.

Dislocation of a joint is a common orthopedic problem. Which of these joints is MOST likely to be dislocated?

Shoulder (glenohumeral) -The articulation between the head of the humerus and the glenoid cavity allows for maximum flexibility but sacrifices stability.

QUESTION 4: Below are boxes showing the events that occur at the NMJ. Drag and drop the boxes into the correct sequence of events. Drag the boxes into the correct sequence where the first box is the first event of the sequence. Ach diffuses across the synaptic cleft Na+ rushed into the sarcolemma to generate an action potential An action potential reaches the synaptic terminal AChE degrades ACh, ending synaptic communication ACh enters the synaptic cleft via exocytosis Ach binds to ACh receptors

Step 1: An action potential reaches the synaptic terminal Step 2: ACh enters the synaptic cleft via exocytosis Step 3: Ach diffuses across the synaptic cleft Step 4: Ach binds to ACh receptors Step 5: Na+ rushed into the sarcolemma to generate an action potential Step 6: AChE degrades ACh, ending synaptic communication

QUESTION 1: The site where a motor neuron excites a skeletal muscle fiber is called the neuromuscular junction. This activity will test your understanding of the sequence of events that occur at the neuromuscular junction. Using the images for help, place in the correct order of occurrence from left to right the events that occur at the neuromuscular junction after the action potential reaches the axon terminal. Membrane potential is less negative. Ca2+ enters the axon terminal. Na+ enters and K+ exits. ACh binds to ACh receptors. Ligand-gated cation channels open. Synaptic vesicles release ACh.

Step 1: Ca2+ enters the axon terminal. Step 2: Synaptic vesicles release ACh. Step 3: ACh binds to ACh receptors. Step 4: Ligand-gated cation channels open. Step 5: Na+ enters and K+ exits. Step 6: Membrane potential is less negative.

Imagine that the cell membrane from the previous problem becomes more permeable to Na+. Predict how this will affect the RMP.

The RMP will be more positive.

Why does regeneration of the action potential occur in one direction, rather than in two directions?

The inactivation gates of voltage-gated Na+‎ channels close in the node, or segment, that has just fired an action potential. -Yes! At the peak of the depolarization phase of the action potential, the inactivation gates close. Thus, the voltage-gated Na+‎ channels become absolutely refractory to another depolarizing stimulus.

QUESTION 3: Match the term in the left column to the blanks near their function on the right. Junctional folds terminal cistern tropomyosin T tubule troponin block(s) binding sites on actin. receive(s) stimulus from the motor neuron. conduct(s) action potentials throughout the interior of the muscle fiber. change(s) shape upon binding with calcium ions. release(s) calcium ions into the sarcoplasm.

Tropomyosin: block(s) binding sites on actin. Junctional folds: receive(s) stimulus from the motor neuron. T tubule: conduct(s) action potentials throughout the interior of the muscle fiber. Troponin: change(s) shape upon binding with calcium ions. Terminal cistern: release(s) calcium ions into the sarcoplasm.

Muscle tissue, one of the four basic tissue groups, consists chiefly of cells that are highly specialized for a. conduction b. cushioning c. contraction d. secretion e. peristalsis

c. contraction

The major role of the sarcoplasmic reticulum is to regulate ________.

intracellular levels of Ca2+

RMP is the:

most important in nerve and muscle cells even though all cells have one

Which order do they occur? 1. Muscle relaxation occurs. 2. ACh molecules are released into the synaptic cleft. 3. Na+ enters the sarcoplasm, producing an action potential. 4. Muscle contraction occurs. 5. The sarcoplasmic reticulum actively pumps back calcium. 6. ACh binds to receptors on the sarcolemma. 7. Calcium ions are released from the sarcoplasmic reticulum. 8. Calcium ions bind to troponin.

2, 6, 3, 7, 8, 4, 5, 1

Put the following events of excitation-contraction coupling in the order in which they occur. 1. Excitation 2. Sarcomere shortening 3. Generation of muscle tension 4. Neural control 5. Contraction cycle begins 6. Release of calcium ions

4. neural control 1. excitation 6. release of calcium ions 5. contraction cycle begins 2. sarcomere shortening 3. generation of muscle tension

Sodium and potassium ions do not diffuse in equal numbers through ligand-gated cation channels. Why?

The inside surface of the sarcolemma is negatively charged compared to the outside surface. Sodium ions diffuse inward along favorable chemical and electrical gradients.

QUESTION 2: Which of the following is/are mechanism(s) to end neural transmission at the neuromuscular junction? Select all the correct answers. a. ACh diffuses away from the synaptic cleft b. ACh is taken up by the axon terminal via endocytosis c. ACh is broken down into acetic acid and choline by the enzyme acetylcholinesterase (AChE) d. ACh binds to ACh receptors

a. ACh diffuses away from the synaptic cleft c. ACh is broken down into acetic acid and choline by the enzyme acetylcholinesterase (AChE)

gluteus medius muscle

abducts, medially rotates thigh; stabilizes pelvis

what actually causes a muscle contraction and relaxation?

* cross bridge cycling * Ca2+ is pumped into the SR (sarcoplasmic reticulum), tropomyosin blockage is restored, and the muscle relaxes

repolarization (action potential)

* immediately after the depolarization wave passes, the sarcolemma permeability changes * Na+ channels close and K+ channels open * K+ diffuses out of the cell, restoring the electrical polarity of the sarcolemma ^^^ this is repolarization

sodium/potassium ATPase pump

* moves 3 Na+ (sodium) out of the cell and 2 K+ (potassium) into the cell for every ATP used * creates concentration gradients that is important to the cell function

multiple motor unit summation

* muscle contracts more vigorously as stimulus strength is increased * this phenomenon, called recruitment, brings more and more muscle fibers respond

nerve stimulus of skeletal muscle

* skeletal muscles are stimulated by motor neurons * each muscle fiber is innervated (connected) by an axon -- the axonal terminal is filled with synaptic vesicles -- the synaptic vesicles contain the neurotransmitter acetylcholine

neuromuscular junction

* the axonal ending and the muscle cell are very close but they do not touch -- the space between the two is called the synaptic cleft -- the muscle cell dips down a bit under the synaptic cleft and is highly fooled THIS IS A MOTOR END PLATE

at low intracellular Ca2+ concentration: roles of ionic calcium (Ca2+) in the contraction mechanism

* tropomyosin blocks the binding sites on action * myosin cross bridges cannot attach to the binding sites on action * the muscle is relaxed

when a nerve impulse reaches the end of an axon at the neuromuscular junction:

* voltage-regulated calcium channels open and allow Ca2+ to enter the axon * Ca2+ inside the axon terminal causes synaptic vesicles to fuse with the axonal membrane and releases ACh into the synaptic cleft via exocytosis

muscle fiber

muscle cell

incomplete tetanus contraction

rapid stimuli, muscle has some time to relax between contractions

conduct(s) action potentials throughout the interior of the muscle fiber

T tubule

At each end of the muscle, the collagen fibers of the epimysium, perimysium, and endomysium, come together to form a a. ligament b. tendon c. tenosynovium d. sheath e. satellite cell

b. tendon

Cl− is a common, negatively charged extracellular ion. Predict the effect on the RMP if many Cl− gated channels are suddenly opened

A more negative RMP would result. -Correct! Cl− is negatively charged and has a higher concentration in the ECF. The opening of Cl− channels would allow more negative charge to attempt to enter the cytoplasm. If the cell's normal RMP were -70mV, it would now become more negative. The farther away the RMP is from zero, either in the positive or in the negative direction, the greater the separation of charges is. This is called hyperpolarization. When the RMP moves closer to 0 mV, depolarization occurs.Congratulations! You've successfully examined the construction of the resting membrane potential, an important concept in any human cell.

Which of the following choices best summarizes excitation-contraction coupling?

A series of events in which an electrical stimulus is conveyed to a muscle fiber to enact contraction -Congratulations! You have completed the final activity in this tutorial. The tutorial covered the sequence of events in excitation-contraction coupling that links the excitation of the membrane to the cross bridge cycling of the contracting proteins. Great job!

A triad is composed of a T-tubule and two adjacent terminal cisternae of the sarcoplasmic reticulum. How are these components connected?

A series of proteins that control calcium release. -Yes! When action potentials propagate along T-tubules, a voltage-sensitive protein changes shape and triggers a different protein to open it's channels, resulting in the release of calcium from the terminal cisternae.

Action potential propagation in a skeletal muscle fiber ceases when acetylcholine is removed from the synaptic cleft. Which of the following mechanisms ensures a rapid and efficient removal of acetylcholine?

Acetylcholine is degraded by acetylcholinesterase.

Rigor mortis occurs due to the lack of what molecule?

ATP

The cross bridge cycle starts when _________.

Ca2+ from the sarcoplasmic reticulum binds to troponin -The release of Ca2+ is triggered by the propagation of an action potential along a skeletal muscle fiber. Ca2+ is released from the sarcoplasmic reticulum and into the sarcoplasm of the muscle fiber. When Ca2+ concentration is high in the sarcoplasm, Ca2+ binds to troponin causing change in its shape. This shape change alters the position of tropomyosin and moves it away from myosin binding sites on actin, thus allowing the myosin head to bind actin and form a cross bridge.

Which selection correctly describes the role of calcium in coupling?

Calcium binds to troponin, which moves tropomyosin and exposes the myosin-binding sites on actin.

People who have hydrocephaly often have an issue with which of these glial cell types?

Ependymal cells -These cells are responsible for the production and circulation of cerebrospinal fluid, and if they are overactive, it can lead to hydrocephaly.

Excitation-contraction coupling is a series of events that occur after the events of the neuromuscular junction have transpired. The term excitation refers to which step in the process?

Excitation, in this case, refers to the propagation of action potentials along the sarcolemma.

Excitation-contraction coupling is a series of events that occur after the events of the neuromuscular junction have transpired. The term excitation refers to which step in the process?

Excitation, in this case, refers to the propagation of action potentials along the sarcolemma. -Yes! These action potentials set off a series of events that lead to a contraction.

The action potential propagates along the sarcolemma. As the action potential spreads down the T tubules of the triads, voltage-sensitive tubule proteins change shape. How does the shape change of these proteins lead to contraction?

It allows calcium to exit the sarcoplasmic reticulum and enter the cytosol. -As the action potential propagates, it changes the shape of T tubule proteins. These proteins are linked to calcium channels in the terminal cisterns of the sarcoplasmic reticulum. When these proteins' calcium channels open, a massive amount of calcium flows into the cytosol.

What is a cross bridge cycle?

It is the cycle in which an energized myosin head binds to actin and performs a power stroke, then binds to ATP in order to detach and re-energize.

the cell membrane is very permeable to: (RMP)

K+ * according to its concentration gradient: K+ wants to diffuse out of the cell

What structure most directly stimulates a skeletal muscle fiber to contract?

Motor neuron -The motor neuron transmits action potentials from the brain or spinal cord to muscle fibers by releasing acetylcholine at the neuromuscular junction.

In which type of axon will velocity of action potential conduction be the fastest?

Myelinated axons with the largest diameter -Yes! The large diameter facilitates the flow of depolarizing current through the cytoplasm. The myelin sheath insulates the axons and prevents current from leaking across the plasma membrane.

QUESTION 4: Match the term in the left column to the blanks near their definition on the right. acetylcholine receptor synaptic vesicle synaptic cleft Sodium channel acetylcholinesterase calcium channel acetylcholine a. a type of voltage-gated ion channel located on the sarcolemma of the muscle fiber. b. a type of voltage-gated ion channel located on the axon terminal. c. a type of chemically-gated ion channel located on the junctional folds of the muscle fiber. d. membranous sac located in the axon terminal that contains neurotransmitter. e. enzyme located in the synaptic cleft that breaks down acetylcholine. f. neurotransmitter that stimulates skeletal muscle contraction. g. the space between the axon terminal and junctional folds.

Sodium channel : a type of voltage-gated ion channel located on the sarcolemma of the muscle fiber. Calcium channel : a type of voltage-gated ion channel located on the axon terminal. Acetylcholine receptor : a type of chemically-gated ion channel located on the junctional folds of the muscle fiber. Synaptic vesicle : membranous sac located in the axon terminal that contains neurotransmitter. Acetylcholinesterase : enzyme located in the synaptic cleft that breaks down acetylcholine. Acetylcholine : neurotransmitter that stimulates skeletal muscle contraction. Synaptic cleft : the space between the axon terminal and junctional folds.

Drag and drop the correct concentrations and the direction of the concentration gradient in a cell at RMP. Answers may be used more than once or not at all. The appropriate arrow should be placed on the plasma membrane—the targets appear on either side of the pumps.

The Na+ concentration is higher outside of the cell, while the concentration of K+ is higher inside the cell.

You are going to record RMP from a cell using an electrode. You place your electrode and record a resting membrane potential every millisecond. You record an initial value of -70mV; however, over time you notice that your recordings become more and more positive until the RMP reaches 0mV. Assuming that Na+ and K+ are the major determinants of RMP in this cell, which of the following could best explain your results?

The cell's Na+-K+ ATPase pumps have stopped functioning. -Yes! Since the RMP eventually becomes zero, the concentration of ions on either side of the membrane would be roughly equal. Without active processes to maintain concentration gradients, we would expect the concentration of ions on either side of the membrane to equilibrate.

Babies are often born with a slightly misshapen head that corrects itself over time. However, in some children suffering from craniosynostosis, the sutures fuse too early and either surgical intervention or corrective helmets must be worn to help correct the problem. Which set of sutures would most likely be affected if the child's head was too short in the anterior-posterior dimension and too tall in the forehead region?

The coronal sutures -These large sutures separating the frontal and parietal bones allow for normal anterior-posterior growth of the infant's skull.

In which of the following would the motor units have the fewest muscle fibers? a. postural muscles of the back b. muscles of the neck c. muscles that control the eyes d. thigh muscles e. calf muscles

c. muscles that control the eyes

A resting neuron is an unstimulated neuron that is not presently generating an action potential. The resting membrane potential is the separation of the relative positive and negative charges across the membrane of a cell at rest. Drag the charges to the correct side of the membrane to represent the resting membrane potential of a typical neuron.

The separation of charges creates a voltage (electrical potential difference), which can be measured using a voltmeter. The resting membrane potential of a neuron averages -70mV (millivolts). All neural activities begin with a change in the resting membrane potential of a neuron.

T/F- In the muscles of the limbs, the origin usually lies proximal to the insertion.

True

What is the relationship between the number of motor neurons recruited and the number of skeletal muscle fibers innervated?

Typically, hundreds of skeletal muscle fibers are innervated by a single motor neuron. -Yes! There are many more skeletal muscle fibers than there are motor neurons. The ratio of neurons to fibers varies from approximately one to ten to approximately one to thousands.

change(s) shape upon binding with calcium ions

troponin

Which type of muscle fiber has a large quantity of glycogen and mainly uses glycolysis to synthesize ATP?

white fast twitch fibers

I band

within the light I band is a dark line called the Z-disc or Z-line

trigger an action potential

you HAVE to release acetylcholine (Ach) to cause the action potential

motor unit: the nerve-muscle functional unit

• A motor unit is a motor neuron and all the muscle fibers it connects to • The number of muscle fibers per motor unit can vary from four to several hundred • Muscles that control fine movements (fingers, eyes) have small motor units • Large weight-bearing muscles (thighs, hips) have large motor units • Muscle fibers from a motor unit are spread throughout the muscle; therefore, contraction of a single motor unit causes weak contraction of the entire muscle

myoglobin

oxygen storing molecules

graded muscle responses

* Graded muscle responses are: -- Variations in the degree of muscle contraction -- Required for proper control of skeletal movement * Responses are graded by: -- Changing the frequency of stimulation -- Changing the strength of the stimulus

"red" and "white" fibers

* Muscle fibers are commonly referred to by color * These also correspond to dark and white meat in poultry -- dark: lots of work, slow to fatigue -- light: not doing much work

Depolarization

* Na+ diffuses into the cell and the interior of the sarcolemma becomes less negative * initially, this is a local electrical event called end plate potential * later, it ignites an action potential that spreads in all directions across the sarcolemma

sarcoplasmic reticulum (SR)

* SR is an elaborate, smooth endoplasmic reticulum * terminal cisternae (lateral sacs) are enlarged areas of the SR * function: store Ca2+ between contractions Ca2+ is critical for contractions

Hyperpolarization: (action potential)

* more K+ diffuses out of the cell, making the membrane more polar than RMP ^^^ this is hyperpolarization * the ionic concentration of the resting state is restored by the Na+ K+ PUMP

oxygen debt

* the amount of oxygen that is required to return the muscle to its pre-exercise state * for a muscle to return to the resting state: -- oxygen reserves must be replenished -- lactic acid must be converted to pyruvic acid -- glycogen stores must be replaced -- ATP and CP reserves must be resynthesized

myofibrils

- densely packed, rodlike contractile elements that run parallel to the length of the cell - makes up most of the muscle volume

skeletal muscle tissue

- has obvious stripes called striations - controlled voluntarily - contracts rapidly but tire easily - responsible for overall motility - is extremely adaptable and can exert forces ranging from a fraction of an ounce to over 70 pounds

attachments of skeletal muscles

- most skeletal muscles span joints and are attached to bone in at least two places - insertion and origin - when muscles contract the movable bone, the muscles insertion, moves toward the immovable bone, the muscles origin

cardiac muscle tissue

- striated but not voluntary - contracts at a fairly steady rate set by the hearts pacemaker - neural controls allow the heart to respond to changes in bodily fluid

structure and organization of skeletal muscle

1) muscle 2) fascicle 3) muscle fiber 4) myofibril or fibril 5) sarcomere 6) myofilament or filament

Muscle functions

1. produce movement: mvmt of whole body, blood, urine, food etc 2. maintain posture: fights gravity to maintain our sitting or standing posture 3. stabilize joints 4. generate heat: heat is a waste product of contractions but is significant source of our stable body temp

Put the following events of the neuromuscular junction in the order in which they occur. 1. Action potential is propagated in the sarcolemma. 2. Acetylcholine binds to ligand gated sodium channels. 3. Action potential arrives at the neuromuscular junction. 4. Vesicles full of acetylcholine are stored at the axon terminal. 5. Acetylcholine is released into the synaptic cleft.

4. vesicles full of acetylcholine 3. action potential arrives at the neuromuscular junction 5. acetylcholine is released into the synaptic cleft 2. acetylcholine binds to ligand gated sodium channels 1. action potential is propagated in the sarcolemma

The resting membrane potential is dependent upon two important factors: 1) differences in sodium and potassium concentrations across the membrane (electrochemical gradients) and 2) differences in sodium and potassium membrane permeability. This activity will help you to review how the resting membrane potential of cells are generated and maintained. Use the provided ions to correctly complete each sentence about the resting membrane potential. Ions may be used more than once, or not at all.

All living cells have a membrane potential that varies depending on its cellular activities.

The binding of the neurotransmitter to receptors on the motor end plate causes which of the following to occur?

Binding of the nuerotransmitter causes chemically gated sodium channels to open in the motor end plate (junctional folds of the sarcolemma) and sodium enters the cell

Which selection correctly describes the role of calcium in coupling?

Calcium binds to troponin, which moves tropomyosin and exposes the myosin-binding sites on actin. -The thin filament in a sarcomere is composed of actin, troponin, and tropomyosin. Troponin and tropomyosin are attached to one another, both overlaying actin. When a muscle is relaxed, tropomyosin blocks actin's myosin-binding sites. Calcium binds to troponin, initiating a shape change that removes the blocking action of tropomyosin. This exposes the myosin-binding sites on actin to the myosin heads for cross bridging.

Excitation of the sarcolemma is coupled or linked to the contraction of a skeletal muscle fiber. What specific event initiates the contraction?

Calcium release from the sarcoplasmic reticulum initiates the contraction.

Excitation of the sarcolemma is coupled or linked to the contraction of a skeletal muscle fiber. What specific event initiates the contraction?

Calcium release from the sarcoplasmic reticulum initiates the contraction. -Yes! Sarcoplasmic reticulum is the specific name given to the smooth endoplasmic reticulum in muscle cells. It is especially abundant and convoluted in skeletal muscle cells. It functions in the storage, release, and reuptake of calcium ions.

What type of conduction takes place in unmyelinated axons?

Continuous conduction -Yes! An action potential is conducted continuously along an unmyelinated axon from its initial segment to the axon terminals. The term continuous refers to the fact that the action potential is regenerated when voltage-gated Na+‎ channels open in every consecutive segment of the axon, not at nodes of Ranvier.

Which of the following effects would EGTA have on excitation-contraction coupling after the neuron releases acetylcholine at the neuromuscular junction?

It would prevent myosin from forming cross bridges with actin.

Which of the following processes produces 36 ATP?

Krebs cycle

QUESTION 3: Drag each tile to the appropriate bin to indicate whether it shows the neuromuscular junction at rest or active (in the process of transmitting a signal).

Neuromuscular junction at rest: 1.) chemically gated channels closed 2.) no Na+ diffusion 3.) calcium channels closed 4.) more negative charges inside sarcolemma 5.) ACh in synaptic vesicles Neuromuscular junction active: 1.) Na+ diffusion 2.) calcium channels open 3.) chemically gated channels open 4.) less negative charges inside sarcolemma 5.) ACh in synaptic cleft

Problems 1-5 view in the assignment

Please study for this exam. I will buy you (myself) lots of starbs and fast food if I study lotssssss bc i know macstudiesalot

Inadequate calcium in the neuromuscular junction would directly affect which of the following processes?

Release of acetylcholine from the synaptic vesicles -Calcium ions enter the axon terminal when voltage-gated calcium channels open in response to the arrival of an action potential. The presence of calcium causes synaptic vesicles to release acetylcholine into the synaptic cleft.

The conversion of an action potential (AP) generated by a motor neuron to contraction of skeletal muscle fiber is called excitation-contraction coupling. This activity will test your understanding of the sequence of events that occur during excitation-contraction coupling. Place the events that occur during excitation-contraction coupling in the correct order from left to right. AP propagates along sarcolemma Sarcoplasmic reticulum releases Ca2+ AP travels down T tubules to triads Ca2+ levels in sarcoplasm increase Voltage-sensitive proteins open Ca2+ channels

Step 1: AP propagates along sarcolemma Step 2: AP travels down T tubules to triads Step 3: Voltage-sensitive proteins open Ca2+ channels Step 4: Sarcoplasmic reticulum releases Ca2+ Step 5: Ca2+ levels in sarcoplasm increase

QUESTION 5: Arrange the sequence of events at the NMJ from first to last. Rank the sequence of events at the NMJ that initiate an action potential in the muscle fiber, from first to last. Action potential is initiated on the sarcolemma Acetylcholine binds to its receptors on the junctional folds. Action potential arrives at the axon terminal. Acetylcholine is released into the synaptic cleft. Calcium ions enter the axon terminal. Synaptic vesicles fuse to membrane of axon terminal. Junctional folds become depolarized.

Step 1: Action potential arrives at the axon terminal. Step 2: Calcium ions enter the axon terminal. Step 3: Synaptic vesicles fuse to membrane of axon terminal. Step 4: Acetylcholine is released into the synaptic cleft. Step 5: Acetylcholine binds to its receptors on the junctional folds. Step 6: Junctional folds become depolarized. Step 7: Action potential is initiated on the sarcolemma

QUESTION 5: Rank the sequence of cross bridge cycling, starting with the myosin-binding sites being exposed and ending with relaxation due to cross bridge cycling ending. Do not overlap any events. Calcium ions pumped into the sarcoplasmic reticulum Myosin head forms cross bridge with actin Myosin binding sites covered Power stroke moves thin filament Cross bridges detach from actin Myosin head is re-energized Calcium ion concentration decreases below the threshold for binding to troponin ATP attaches to myosin head

Step 1: Myosin head forms cross bridge with actin Step 2: Power stroke moves thin filament Step 3: ATP attaches to myosin head Step 4: Cross bridges detach from actin Step 5: Myosin head is re-energized Step 6: Calcium ions pumped into the sarcoplasmic reticulum Step 7: Calcium ion concentration decreases below the threshold for binding to troponin Step 8: Myosin binding sites covered

The sarcomere shortens when the myosin heads of the thick filaments, in a cocked position, form cross bridges with the actin molecules in thin filaments. This activity will test your understanding of the steps that occur in one complete cross bridge cycle. Place the steps that occur during a single cross bridge cycle in the correct order from left to right. ATP binds to the myosin head and detaches it from actin. The activated myosin head binds to actin,forming a cross bridge. ATP is hydrolyzed to ADP and Pi and the energy released re-cocks the myosin head. ADP is released and myosin slides the thin filament toward the center of the sarcomere.

Step 1: The activated myosin head binds to actin,forming a cross bridge. Step 2: ADP is released and myosin slides the thin filament toward the center of the sarcomere. Step 3: ATP binds to the myosin head and detaches it from actin. Step 4: ATP is hydrolyzed to ADP and Pi and the energy released re-cocks the myosin head.

QUESTION 5: Rank the sequence of events in excitation-contraction coupling from first to last. Do not overlap any events. Tropomyosin moves off of actin binding sites. Depolarization of the junctional folds triggers an action potential. Calcium ions flood the sarcoplasm. Myosin heads form crossbridges with actin. Calcium ions bind to troponin. Troponin changes shape. Action potential travels along the sarcolemma. Action potential along the T tubules opens calcium channels.

Step 1:Depolarization of the junctional folds triggers an action potential. Step 2: Action potential travels along the sarcolemma. Step 3: Action potential along the T tubules opens calcium channels. Step 4: Calcium ions flood the sarcoplasm. Step 5: Calcium ions bind to troponin. Step 6: Troponin changes shape. Step 7: Tropomyosin moves off of actin binding sites. Step 8: Myosin heads form crossbridges with actin.

Calcium entry into the axon terminal triggers which of the following events?

Synaptic vesicles fuse to the plasma membrane of the axon terminal and release acetylcholine.

What is the function of the myelin sheath?

The myelin sheath increases the speed of action potential conduction from the initial segment to the axon terminals. -Yes! The myelin sheath increases the velocity of conduction by two mechanisms. First, myelin insulates the axon, reducing the loss of depolarizing current across the plasma membrane. Second, the myelin insulation allows the voltage across the membrane to change much faster. Because of these two mechanisms, regeneration only needs to happen at the widely spaced nodes of Ranvier, so the action potential appears to jump.

Myasthenia gravis is a disease resulting from an autoimmune attack on the ACh receptors of the motor end plate. Binding of antibodies to the ACh receptors results in generalized muscle weakness that progresses as more ACh receptors are destroyed. Which of the following medications would help alleviate the muscle weakness?

a drug that binds to and inactivates acetylcholinesterase (neostigmine) -The progressive destruction of ACh receptors leads to a progressive decline in the strength of end-plate potentials. To counteract this problem, drug therapy focuses on increasing the concentration of acetylcholine in the synaptic cleft. Drugs that bind to acetylcholinesterase and prevent the breakdown of acetylcholine can help restore some muscle strength by allowing acetylcholine to interact with the ACh receptors longer. Neostigmine is one such drug and is used to treat myasthenia gravis.

The response of a motor unit to a single action potential of its motor neuron is called ________.

a muscle twitch

The region of the sarcomere containing the thick filaments is the a. A band b. M line c. H band d. Z line e. I band

a. A band

Which of the following statements about excitation-contraction coupling is incorrect? a. Calcium ions travel through the transverse tubule b. Troponin binds calcium ion and signals tropomyosin to move c. Tropomyosin moves to expose myosin binding sites on actin d. Relaxation requires uptake of calcium ion by the sarcoplasmic reticulum e. Calcium ion is released from the sarcoplasmic reticulum

a. Calcium ions travel through the transverse tubule

QUESTION 2: What structure most directly stimulates a skeletal muscle fiber to contract? a. Motor neuron b. Voltage-gated calcium channels c. Synaptic cleft d. Acetylcholinesterase

a. Motor neuron

Which of the following statements is true concerning motor units? a. Movement of the eyeball requires fewer muscle fibers with less tension being produced b. A motor unit consists of all the muscle fibers it controls as well as the area of the spinal cord where it originates c. Lesser tension is required in lifting a dumbbell than in lifting a cup of coffee d. An increased number of motor neurons is needed in precise movements such as lifting a cup of coffee than in lifting a dumbbell e. A small number of muscle fibers with increased tension is needed in large, gross movements

a. Movement of the eyeball requires fewer muscle fibers with less tension being produced

Identify the incorrect statement about a single motor unit. a. The more neurons involved, the more powerful the contraction b. Fine motor skills depend on the development of small motor units c. The smaller the number of muscle fibers, the more precise the movement d. Some motor units include as many as 2000 muscle fibers e. Muscle fibers of one motor unit intermingle with the fibers of another motor unit

a. The more neurons involved, the more powerful the contraction

QUESTION 2: Why do the disease tetanus and the normal sustained muscle contraction called tetanus share the same name? a. They both eliminate the relaxation phase in the affected muscle fibers b. They are both brought on by bacterial infections c. They both enable muscle fibers to reach near maximum tension d. All of the listed responses are correct.

a. They both eliminate the relaxation phase in the affected muscle fibers

Active sites on the actin become available for binding after a. calcium binds to troponin b. calcium binds to tropomyosin c. actin binds to troponin d. myosin binds to troponin e. troponin binds to tropomyosin

a. calcium binds to troponin

The type of contraction where the tension is less than the load is called a. isotonic eccentric contraction b. isometric eccentric contraction c. isometric contraction d. isotonic concentric contraction e. isometric concentric contraction.

a. isotonic eccentric contraction

What is the function of the structure indicated by the arrow? (question 37) a. making of energy (ATP); "power house" of the cell b. distribute action potentials throughout the interior of the skeletal muscle cell c. houses the genetic material (DNA) of the cell d. storage of calcium

a. making of energy (ATP); "power house" of the cell

The advantage of having many nuclei in a skeletal muscle fiber is the ability to a. produce large amounts of muscle proteins b. contract much more forcefully c. produce nutrients for muscle contraction d. store extra DNA for metabolism e. produce more ATP with little oxygen

a. produce large amounts of muscle proteins

Which of the following best describes the term sarcoplasmic reticulum? a. storage and release site for calcium ions b. protein that accounts for elasticity of resting muscle c. repeating unit of striated myofibrils d. largely made of myosin molecules e. thin filaments are anchored here

a. storage and release site for calcium ions

The narrow space between the synaptic terminal and the muscle fiber is the a. synaptic cleft b. motor end plate c. synaptic knob d. M line e. motor unit

a. synaptic cleft

In the sarcomere which elastic protein attaches the thick filament to the Z line? a. titin b. actin c. G actin d. nebulin e. myosin

a. titin

Each thin filament consists of a. two actin protein strands coiled helically around each other b. a rod-shaped structure with "heads" projecting from each end c. a double strand of myosin molecules d. chains of myosin molecules e. six molecules coiled into a helical structure

a. two actin protein strands coiled helically around each other

QUESTION 5: Which type of muscle fiber has a large quantity of glycogen and mainly uses glycolysis to synthesize ATP? a. white fast twitch fibers b. red slow twitch fibers

a. white fast twitch fibers

In a sarcomere, cross-bridge attachment occurs specifically in the a. zone of overlap b. H band c. A band d. M line e. I band

a. zone of overlap

The region of sarcomere where thin and thick filaments are located is called the a. zone of overlap b. Z line c. I band d. M line e. A band

a. zone of overlap

When a skeletal muscle fiber contracts, the a. zones of overlap get larger b. width of the A band increases c. H bands and I bands get larger d. Z lines move further apart e. All of the answers are correct

a. zones of overlap get larger

In a neuromuscular junction, synaptic vesicles in the motor neuron contain which neurotransmitter?

acetylcholine (ACh)

calcium breaks down ______ in the axonal terminal?

ach

A myosin head binds to whihc molecule to form a cross bridge?

actin

The striated appearance of skeletal muscle results from

actin and myosin arrangement

Force of Muscle Contraction

affected by 3 things: 1) the number of muscle fibers contracting: the more motor fibers in the muscle, the stronger the contraction 2) the relative size of the fibers: the bulkier the muscle, the greater its strength 3) degree of muscle stretch: muscles contract strongest when muscle fibers are 80-120% of their normal resting length

Muscles whose actions oppose one another are referred to as

antagonists

Yes! The large diameter facilitates the flow of depolarizing current through the cytoplasm. The myelin sheath insulates the axons and prevents current from leaking across the plasma membrane.

anterograde transport -This transport mechanism defines movement of material from the cell body (soma) of a neuron toward the axon terminals (synaptic knobs).

How would the loss of acetylcholinesterase from the motor end plate affect skeletal muscle? a. It would make the muscles less excitable b. It would cause muscles to stay contracted c. It would produce muscle weakness d. It would cause muscles to stay relaxed e. It would have little effect on skeletal muscles

b. It would cause muscles to stay contracted

The skeletal muscle complex known as the triad consists of a. A bands, H bands, and I bands b. a transverse tubule and two terminal cisternae c. actin, myosin, and titin filaments d. actin, myosin, and sarcomeres e. filaments, myofibrils, and sarcomeres

b. a transverse tubule and two terminal cisternae

Communication between axons and muscle fibers occurs at specialized synapses called a. nervous units b. neuromuscular junctions c. synaptic terminals d. motor end plates e. motor units.

b. neuromuscular junctions

QUESTION 6: Acetylcholine receptors are primarily located __________. a. inside the muscle fiber b. on the motor end plate c. inside vesicles d. on the synaptic terminal

b. on the motor end plate

QUESTION 8: The role of acetylcholinesterase in the neuromuscular junction is to __________. a. increase the sodium permeability of the motor end plate b. remove acetylcholine from the synaptic cleft c. generate a muscle action potential d. release acetylcholine from the synaptic terminal

b. remove acetylcholine from the synaptic cleft

All of the following proteins are part of the thin filaments except a. tropomyosin b. titin c. actin d. troponin e. None of the answers is correct; there are no exceptions

b. titin

At rest, the tropomyosin molecule is held in place by a. actin molecules b. troponin molecules c. myosin molecules d. calcium ions e. ATP molecules

b. troponin molecules

The "rest and recovery" period, where the muscle restores depleted reserves, includes all of the following processes EXCEPT __________. a) Oxygen rebinds to myoglobin. b) ATP is used to rephosphorylate creatine into creatine phosphate. c) Pyruvic acid is converted back to lactic acid. d) Glycogen is synthesized from glucose molecules

c) Pyruvic acid is converted back to lactic acid.

QUESTION 1: Tamara's muscle weakness and fatigue becomes progressively worse over the course of the day. This is a hallmark sign of myasthenia gravis, an autoimmune disorder that affects the ability of a motor neuron to communicate with a muscle fiber. What synaptic events must happen first for excitation to occur? a. The action potential travels down the T tubules in the sarcolemma and triggers the release of calcium ions from the sarcoplasmic reticulum b. Calcium binds to tropomyosin, shifting it from the active site on actin, allowing cross-bridge formation to occur as myosin binds to the active site c. Acetylcholine is released from the axon terminal and diffuses across the synapse to bind to a receptor in the surface of the motor end plate d. Sodium ions from the extracellular fluid diffuse into the cytosol of the muscle fiber, triggering the generation of an action potential in the sarcolemma

c. Acetylcholine is released from the axon terminal and diffuses across the synapse to bind to a receptor in the surface of the motor end plate

QUESTION 3: The injection that dramatically improved Tamara's symptoms was Tensilon, an acetylcholinesterase inhibitor. What is acetylcholinesterase, and why was this injection effective in relieving Tamara's symptoms? a. Acetylcholinesterase is an enzyme that degrades the ACh receptors. When acetylcholinesterase is inhibited, there are more receptors available for acetylcholine to bind, and muscle function improves b. Acetylcholinesterase is an acetylcholine antagonist that binds to motor end plate receptors and decreases the permeability of the sarcolemma to sodium. Inhibiting acetylcholinesterase increases the influx of sodium, leading to the generation of action potentials and an improvement in muscle function c. Acetylcholinesterase is the enzyme that clears acetylcholine from the synapse. Inhibiting this enzyme will enable acetylcholine to be available for a longer period to bind to receptors d. Acetylcholinesterase is a regulatory enzyme that inhibits the ACh receptors on the motor end plate. Inhibiting acetylcholinesterase improves muscle function because the ACh receptors are temporarily more active

c. Acetylcholinesterase is the enzyme that clears acetylcholine from the synapse. Inhibiting this enzyme will enable acetylcholine to be available for a longer period to bind to receptors

QUESTION 2: Myasthenia gravis is an autoimmune disorder that results in the production of antibodies that either block or cause the destruction of the ACh receptor. Why do you think that Tamara's symptoms become worse as the day progresses? a. In myasthenia gravis the ACh receptors have a hyperaffinity to acetylcholine. After muscle activity, fatigue and weakness occur as fewer unbound receptors are available to bind acetylcholine b. In the morning, the reserves of ATP are larger, allowing for sufficient acetylcholine release to overcome the unresponsive receptors. However, as the day progresses and stores of ATP become depleted, muscle function becomes progressively worse c. In the morning, the neuronal reserves of acetylcholine are larger, allowing for sufficient neurotransmitter release and resulting in effective muscle signaling. As the day progresses and the stores of acetylcholine become depleted, muscle function becomes progressively worse. d. As the muscle becomes fatigued over the course of the day's activities, the receptors bind less acetylcholine, which results in smaller and smaller action potentials being generated. Smaller action potentials mean weaker contractions, as seen by the increasing muscle weakness.

c. In the morning, the neuronal reserves of acetylcholine are larger, allowing for sufficient neurotransmitter release and resulting in effective muscle signaling. As the day progresses and the stores of acetylcholine become depleted, muscle function becomes progressively worse.

QUESTION 3: Which of the following processes produces 36 ATP? a. hydrolysis of creatine phosphate b. glycolysis c. Krebs cycle and oxidative phosphorylation

c. Krebs cycle and oxidative phosphorylation

The cytoplasm of the neuromuscular terminal contains vesicles filled with molecules of the neurotransmitter a. adrenaline b. antidiuretic hormone c. acetylcholine d. norepinephrine e. epinephrine

c. acetylcholine

What is the function of the structure indicated by the arrow? (question 35) a. making of energy (ATP); "power house" of the cell b. part of coupling the action potential to contraction c. houses the genetic material (DNA) of the cell d. storage of calcium

c. houses the genetic material (DNA) of the cell

Which of the following is not an effect of aging on the muscular system? a. smaller muscle fibers b. decreased recovery from muscle injuries c. hypertrophy d. exercise tolerance decreases e. muscles become less elastic

c. hypertrophy

Heat energy gained from muscle contraction is released by the ________ system. a. respiratory b. cardiovascular c. integumentary d. urinary e. endocrine

c. integumentary

The type of contraction where the tension exceeds the load is called a. isometric concentric contraction b. isometric contraction c. isotonic concentric contraction d. isometric eccentric contraction e. isotonic eccentric contraction.

c. isotonic concentric contraction

In an isotonic contraction, a. postural muscles stabilize the vertebrae b. the peak tension is less than the load c. muscle tension exceeds the load and the muscle lifts the load d. many twitches always fuse into one e. tension rises and falls but the muscle length is constant.

c. muscle tension exceeds the load and the muscle lifts the load

A patient takes a medication that blocks ACh receptors of skeletal muscle fibers. What is this drug's effect on skeletal muscle contraction? a. increases tone in the muscle b. produces a strong, continuous state of contraction c. reduces the muscle's ability for contraction d. causes a strong contraction similar to a "charlie horse" cramp e. increases the muscle's excitability

c. reduces the muscle's ability for contraction

The type of muscle fiber that is most resistant to fatigue is the ________ fiber. a. intermediate b. fast c. slow d. high-density e. anaerobic

c. slow

Which of the following statements correctly describes the structure (band or line) of the sarcomere indicated by the arrow? (question 39) a. the region of the resting sarcomere that only contains thick filaments b. the point of connection for adjacent thick filaments c. the boundary between adjacent sarcomeres d. the region of the sarcomere that contains only thin filaments

c. the boundary between adjacent sarcomeres

Which of the following statements correctly describes the structure (band or line) of the sarcomere indicated by the arrow? (question 40) a. the region of the sarcomere that contains only thin filaments b. the region of the resting sarcomere that only contains thick filaments c. the point of connection for adjacent tails of the thick filaments d. the boundary between adjacent sarcomeres

c. the point of connection for adjacent tails of the thick filaments

At rest, active sites on the actin are blocked by a. ATP molecules b. calcium ions c. tropomyosin molecules d. myosin molecules e. troponin molecules

c. tropomyosin molecules

The rapid rise and fall in force produced by a muscle fiber after a single action potential is a(n) a. unfused tetanus b. muscle action potential c. twitch d. tetanus e. motor end plate potential

c. twitch

Which of the following is most directly responsible for the coupling of excitation to contraction of skeletal muscle fibers?

calcium ions

external oblique muscle

compresses abdomen; rotates trunk

internal oblique muscle

compresses abdomen; rotates trunk

rectus abdominis muscle

compresses abdomen; rotates trunk

transverse abdominis muscle

compresses abdomen; rotates trunk

fast glycolytic fibers

contract quickly, have fast myosin ATPase, and are easily fatigued

fast oxidative fibers

contract quickly, have fast myosin ATPases, and have moderate resistance to fatigue

slow oxidative fibers

contract slowly, have slow acting myosin ATPases, and are fatigue resistant

sarcoplasm

cytoplasm of a muscle cell

QUESTION 1: The cross bridge cycle starts when _________. Select the best answer. a. Ca2+ is actively transported into the sarcoplasmic reticulum b. Ca2+ from the sarcoplasmic reticulum binds to tropomyosin c. acetylcholine diffuses away from the synaptic cleft d. Ca2+ from the sarcoplasmic reticulum binds to troponin e. ATP binds to troponin and is hydrolyzed to ADP and Pi

d. Ca2+ from the sarcoplasmic reticulum binds to troponin

QUESTION 1: Which of the following is responsible for muscle relaxation? a. ATP binds to the myosin head to cause cross bridge detachment. b. Calcium ions are released from the terminal cisterns during depolarization of the T tubule c. Sodium ions enter into the sarcoplasm through voltage-gated channels d. Calcium ions are removed from the sarcoplasm by active transport

d. Calcium ions are removed from the sarcoplasm by active transport.

QUESTION 2: Which of the following is NOT a role of ATP in muscle contraction? a. Transporting calcium ions into the sarcoplasmic reticulum b. Energizing the power stroke of the cross bridge c. Detaching the cross bridge from actin d. Exposing myosin binding sites on actin

d. Exposing myosin binding sites on actin

QUESTION 4: The "rest and recovery" period, where the muscle restores depleted reserves, includes all of the following processes EXCEPT __________. a. Oxygen rebinds to myoglobin b. Glycogen is synthesized from glucose molecules c. ATP is used to rephosphorylate creatine into creatine phosphate d. Pyruvic acid is converted back to lactic acid

d. Pyruvic acid is converted back to lactic acid

Which of the following is important to increasing efficiency in tension production? a. increasing calcium reclamation b. lengthening the zone of overlap c. increasing the latent period d. a shortened relaxation phase

d. a shortened relaxation phase

Which of the following situations could contribute to prolonged muscle contraction? a. limited availability of calcium ions b. infrequent neural stimulus c. cross-bridge formation being reduced d. acetylcholinesterase not being produced

d. acetylcholinesterase not being produced

The most important factor in decreasing the intracellular concentration of calcium ion after contraction is a. active transport of calcium into the synaptic cleft b. diffusion of calcium into the sarcoplasmic reticulum c. active transport of calcium across the sarcolemma d. active transport of calcium into the sarcoplasmic reticulum e.diffusion of calcium out of the cell

d. active transport of calcium into the sarcoplasmic reticulum

Creatine phosphate a. is only formed during strenuous exercise b. can replace ATP in binding to myosin molecules during contraction c. cannot transfer its phosphate group to ADP d. acts as an energy reserve in muscle tissue e. is produced by the process of anaerobic respiration.

d. acts as an energy reserve in muscle tissue

The point in a muscle twitch when the troponin is bound to calcium is called the a. isotonic period b. stimulus phase c. relaxation phase d. contraction phase e. latent period

d. contraction phase

Physical evidence that supports the sliding filament theory of muscle contraction includes a. the I band + H band distance is constant during contraction b. increased width of the I band during contraction c. decreased width of the A band during contraction d. decreased width of the H band during contraction e. constant distance between Z lines during contraction

d. decreased width of the H band during contraction

QUESTION 2: Conduction of an action potential along the sarcolemma depends upon ___________. a. shifting tropomyosin to uncover myosin binding sites b. binding of acetylcholine to chemically gated channels c. release of calcium ions from the terminal cisterns d. diffusion of sodium ions through voltage-gated channels

d. diffusion of sodium ions through voltage-gated channels

Nerves and blood vessels that service a muscle fiber are located in the connective tissues of its a. myofibrils b. sarcomere c. perimysium d. endomysium e. sarcolemma

d. endomysium

A weight-lifter strains to lift a heavy weight and there is no movement of the person's arms holding on to the weight. This type of contraction is called a(n) ________ contraction. a. isotonic b. tetanus c. concentric d. isometric e. treppe

d. isometric

A single motor neuron together with all the muscle fibers it innervates is called a(n) a. end plate b. end foot c. dermatome d. motor unit e. myotome

d. motor unit

Interactions between actin and myosin filaments of the sarcomere are responsible for a. the striped appearance of skeletal muscle b. the conduction of neural stimulation to the muscle fiber c. muscle relaxation d. muscle contraction e. muscle fatigue.

d. muscle contraction

Which of the following statements correctly describes the structure (band or line) of the sarcomere indicated by the arrow? (question 38) a. the boundary between adjacent sarcomeres b. the region of the sarcomere that contains only thin filaments c. the point of connection for adjacent thick filaments d. the region of the resting sarcomere that only contains thick filaments

d. the region of the resting sarcomere that only contains thick filaments

Which of the following best describes the term Z line? a. storage site for calcium ions b. largely made of myosin molecules c. repeating unit of striated myofibrils d. thin filaments are anchored here e. protein that accounts for elasticity of resting muscle

d. thin filaments are anchored here

QUESTION 4: Inside a neuron, acetylcholine is contained within __________. a. the motor end plate b. the synaptic cleft c. acetylcholine receptors d. vesicles

d. vesicles

If a second stimulus arrives before the relaxation phase has ended, a second, more powerful contraction occurs. This addition of one twitch to another is called a. complete tetanus b. incomplete tetanus c. treppe d. wave summation e. recruitment

d. wave summation

a cell becoming less negative =

depolarization

A resting muscle generates most of its ATP by a. hydrolysis of creatine phosphate b. glycogenolysis c. anaerobic respiration d. the tricarboxylic acid cycle e. aerobic metabolism of fatty acids

e. aerobic metabolism of fatty acids

The structural theory that explains how a muscle fiber contracts is called the ________ theory. a. action-myosin interaction b. neuromuscular c. muscle contraction d. excitation-contraction coupling e. sliding filament

e. sliding filament

The bundle of collagen fibers at the end of a skeletal muscle that attaches the muscle to bone is called a(n) a. fascicle b. epimysium c. ligament d. myofibril e. tendon

e. tendon

Which of the following become connected by myosin cross-bridges during muscle contraction? a. thick filaments and titin filaments b. thick filaments and t-tubules c. thin filaments and t-tubule d. z disks and actin filaments e. thin filaments and thick filaments

e. thin filaments and thick filaments

Action potentials at the sarcolemma are conducted into the inside of a skeletal muscle fiber by a. sarcoplasmic reticulum b. triads c. motor end plates d. neuromuscular junctions e. transverse tubules

e. transverse tubules.

A muscle that is lengthening while it produces tension is performing a(an) ________ contraction.

eccentric

Which muscle characteristic describes the ability of muscle to respond to a stimulus?

excitability

What means of membrane transport is used to release the neurotransmitter into the synaptic cleft?

exocytosis

Action potentials travel the length of the axons of motor neurons to the axon terminals. These motor neurons __________.

extend from the brain or spinal cord to the sarcolemma of a skeletal muscle fiber

triceps brachii muscle

extends and adducts forearm

vastus lateralis muscle

extends and stabilizes knee

vastus medialis muscle

extends leg at knee

rectus femoris muscle

extends leg at knee; flexes leg at knee

biceps femoris muscle (hamstrings, quadriceps)

extends thigh at hip, flexes leg at knee

gluteus maximus muscle

extends thigh, laterally rotates and abducts thigh

latissimus dorsi muscle

extends, adducts, and rotates arm medially

latent period of muscle twitch

first few milliseconds after stimulation when excitation-contraction coupling is taking place (everything between stimulus and actin and myosin binding)

pectoralis major muscle

flexes, adducts and rotates arm

Unlike skeletal tissue, cardiac muscle tissue

is involuntary, and contains branched cells and intercalated discs

receive(s) stimulus from the motor neuron

junctional folds

When muscle cells break down glucose to generate ATP under oxygen deficient conditions, they will form ________.

lactic acid

Part complete You are working on a new medication to selectively kill cancer cells. You test the drug on a culture of normal neurons to check for any side effects. Despite the presence of the drug, a normal resting membrane potential of −70 mV is maintained. Look at each of the figures below to analyze how the drug might be affecting the neuron. Figure A shows neuron activity under normal, or control, conditions (without the drug). Figure B shows neuron activity that has been altered due to exposure to the drug. Based on these results, the drug is most likely __________.

preventing Na+-gated channels from opening

Number of cross-bridges decline and tension is reduced during which phase?

relaxation phase

What causes the power stroke?

release of ADP and Pi

Excitation-contraction coupling includes all EXCEPT which of the following events? - binding of calcium ions to troponin, which removes the blocking action of tropomyosin -propagation of an action potential along the sarcolemma and down T tubules -release of calcium ions from the terminal cisterns -release of acetylcholine from axon terminals at the neuromuscular junction

release of acetylcholine from axon terminals at the neuromuscular junction

cross bridge cycle

sequence of events between binding of a cross-bridge to actin, its release, and reattachment during muscle contraction

Aponeurosis

sheets of connective tissue

Which joint has most notably sacrificed stability to provide great freedom of movement?

shoulder -In the shoulder joint, stability has been sacrificed to provide the most freely moving joint in the body. The articulating bones provide minimal joint stability because of the size and "fit" of the articulating surfaces. The major stabilizing forces are soft tissue, in particular the tendons of the rotator cuff muscles (via muscle tone).

How is acetylcholine (ACh) removed from the synaptic cleft?

simple diffusion away from the synaptic cleft and acetylcholinesterae (AChE; an enzyme)

Which type of muscle CANNOT contract without being stimulated by the nervous system?

skeletal

3 types of muscle tissue

skeletal, cardiac, smooth

What special feature of smooth muscle allows it to stretch without immediately resulting in a strong contraction?

stress-relaxation response

maximum contraction or response

strongest contraction that a muscle can have

release(s) calcium ions into the sarcoplasm

terminal cistern

load

the opposing force on the muscle ex: the weight of the object to be moved

membrane potential is created by: (RMP)

the position of ions relative to the cell membrane

If a signal from a sending neuron makes the receiving neuron more negative inside,

the receiving neuron is less likely to generate an action potential. -If the receiving neuron is more negative inside, it is less likely to generate an action potential.

The binding of calcium to which molecule causes the myosin binding sites to be exposed?

troponin

Rigor mortis refers to the changes in muscle tissue that occur after death. The muscles become very stiff or rigid and are frozen in a contracted state. Rigor mortis usually sets in within four hours, first in the face and generally smaller muscles. The body reaches maximum stiffness in twelve to forty-eight hours. However, this time may vary due to environmental conditions, with cooler conditions inhibiting rigor mortis. Which of the following images would depict the state of the cross bridge cycle during rigor mortis?

view image in question 14

Destruction of Acetylcholine

* ACh bound to ACh receptors is quickly destroyed by the enzyme acetylcholinesterase * This destruction prevents continued muscle fiber contraction in the absence of additional stimuli

neuromuscular junction con't

* ACh diffuses across the synaptic cleft to ACh receptors on the sarcolemma * binding of ACh to its receptors initiates an action potential in the muscle

roles of acetylcholine (Ach) w/ depolarization

* Ach binds to its receptors at the motor end plate * binding opens chemically (ligand) gated sodium channels * Na+ diffuses into the cell and the interior of the sarcolemma becomes less negative ^^^this is known as depolarization

depolarization and generation of an action potential: (action potential)

* Ach from the neuron binds to receptors in the motor end plate and causes sodium channels to open * Na+ flows into the cell * inside of the cell becomes positive

T tubules

* T tubules are projections of the sarcolemma that occur at every junction of an A band and I band * 1 T tubule plus 2 terminal cisternae = triad

ultrastructure of myofilaments: Thick filaments

* Thick filaments are composed of the protein myosin. * Each myosin molecule has a rod-like tail and two globular heads - Tails-interwoven to form a fiber - Heads * looks like two golf clubs wrapped together * each thick filament contains approximately 200 molecules of myosin

at higher intracellular Ca2+ concentrations: roles of ionic calcium (Ca2+) in the contraction mechanism

* additional calcium binds to troponin * calcium-activated troponin undergoes a conformational (shape) change * troponin pulls tropomyosin away from actin's binding site * myosin head can now bind and cycle * this permits contraction (sliding of the thin filaments by the myosin cross bridge) to begin

propagation of an action potential: (action potential)

* change in the charge causes voltage gated Na+ channels to open * Na+ flows into the cell in those areas ^^^ this is now depolarization * do a "wave" * once initiated, the action potential is unstoppable, and ultimately results in the contraction of a muscle

sliding filament model of contraction

* when a muscle contracts, thin filaments slide past the thick ones so that the actin and myosin filaments overlap to a great degree -- distance between z discs is reduced -- I band shorten -- H zone disappear * in the relaxed state, thin and thick filaments overlap only slightly

myofilaments: Z-disc

- coin-shaped sheet of proteins (connections) that anchors the thin filaments

smooth muscle tissue

- in walls of hollow visceral organs, like the stomach, bladder, respiratory system - forces food and other substances thru internal body channels - it is not striated and is involuntary

a sarcomere is:

- the smallest contractile unit of a muscle - runs z-disc to z-disc

sarcomere structure

- thin (actin) filament - elastic (titin) filament - thick (myosin) filament

myofilaments: H-zone

- thin filaments do not overlap thick filaments in the lighter H-zone - H-zone contains thick filaments only

A band

- within the A band, there is a lighter H zone that appears in relaxed muscle - in the middle of the H zone is a dark M line

roots for muscles

-myo -mys -sarco

sequential events of contraction

1) Cross bridge formation: myosin cross bridge attaches to actin filament 2) power stroke: myosin head pivots and pulls actin filament toward M line 3) Cross bridge detachment: new ATP attaches to myosin head and the cross bridge detaches 4) "Cocking" of the myosin head: energy from hydrolysis of ATP cocks the myosin head into the high energy state

excitation-contraction coupling

1) action potential is generated and propagated along the sarcolemma to the T-tubules 2) action potential triggers Ca2+ release 3) Ca2+ bind to troponin, blocking action of tropomyosin release 4) contraction via crossbridge formation 5) removal of Ca2+ by active transport 6) tropomyosin blockage restored, contraction ends

period of relaxation of muscle twitch

Ca2+ is pumped back into the SR; actin and myosin separate; muscle lengthens; muscle tension returns to zero

stimulus intensity and muscle tension

it doesn't matter how much we increase the stimulus, the maximum contraction will NOT go up

synaptic cleft

gap between the axonal terminal (presynaptic) and the muscle cell (postsynaptic)

RMP evenly distributed =

if there were only passive processes, eventually Na+ and K+ would be equally distributed across the membrane

the inside of the cell is _________ relative to the outside

negative * even though we are talking about positive ions, we are looking at relative charge


Related study sets

Chapter 28 Head and Spine Injuries

View Set

Social Psych Textbook Chapters 1-5, Social psych exam 4, Social psych exam 3, Social psych Exam 2, Social Psych exam 1

View Set

The Greenhouse Effect and Increasing Greenhouse Gases

View Set

Health Assessment Ch 15 Head and Neck

View Set